1
$\begingroup$

Let $p$ be a prime number and $l$ be the greatest prime number less than $2p$. Moreover, let $m,t_i<p$ be positive integer numbers such that $\sum_{i=1}^mit_i=2p$. Is it possible that $1^{t_1}2^{t_2}\ldots m^{t_m}t_1!t_2!\ldots t_m!\mid(2p-l)!$

$\endgroup$
4
  • $\begingroup$ Did you mean $\ell !& or something? $2p-1$ could be prime. $\endgroup$
    – alpoge
    Aug 9, 2015 at 21:17
  • $\begingroup$ @alpoge, I take the question to be, do there exist $p$, $m$, and $t_i$ such that .... You are pointing out that if $2p-1$ is prime then $m$ and $t_i$ don't exist, but what if $2p-1$ is not prime? $\endgroup$ Aug 10, 2015 at 0:14
  • $\begingroup$ @GerryMyerson the question asks if for some prime $p$ we express him as $\sum i\cdot t_i$ ....then is it possible that $1^{t_1}...t_m!\mid (2p-l)!$ One example can answer the question in a positive way. $\endgroup$ Aug 10, 2015 at 7:24
  • 6
    $\begingroup$ @nina The number-theory tag already indicates that your question is about number theory. In future, it would be better to use a more descriptive title. $\endgroup$ Aug 10, 2015 at 11:42

1 Answer 1

8
$\begingroup$

It is possible. Take $p=61$ and $t_1=2$, $t_{120}=1$. Then $l=113$ and $1^{t_1}\cdot t_1!\cdot 120^{t_{120}}\cdot t_{120}!=240$ divides $(2p-l)!=9!=362880$.

Another example: $p=673$, $t_1=t_{672}=2$. Then $l=1327$ and $1^{t_1}\cdot t_1!\cdot 672^{t_{672}}\cdot t_{672}! = 1806336$ divides $(2p-l)!=19!$.

UPDATE. If all $t_1,\dots,t_m$ are required to be (strictly) positive, then there are likely no required examples. Numerically I tested this for primes $p$ below $10^5$. For large $p$ this can be assessed heuristically as follows.

By Cramer's conjecture, $2p-l$ is asymptotically bounded by $\log(p)^2$. We also have that $m!$ divides $1^{t_1}\cdot 2^{t_2}\cdots m^{t_m}$, which in turn divides $(2p-l)!$, and thus $m\leq 2p-l$. Furthermore, since $\sum_{i=1}^m it_i=2p$, at least one of $t_i$ must be at least $$\frac{2p}{1+2+\dots+m}\geq \frac{4p}{(m+1)^2}\geq \frac{4p}{(2p-l+1)^2}.$$ That is, $t_1!\cdots t_m!\mid (2p-l)!$ would asymptotically imply $\frac{4p}{\log(p)^4}\leq \log(p)^2$, which is not possible.

$\endgroup$
2
  • 2
    $\begingroup$ But the $t_i$ are supposed to be positive integers for $i$ from $1$ to $m$. If $t_{120}=1$, then $t_2,t_3,\dots,t_{119}$ have to be positive integers as well, and then $\sum it_i\dots$. $\endgroup$ Aug 10, 2015 at 13:29
  • $\begingroup$ @GerryMyerson: Good point (if the "positive" is not in French meaning). For primes $p$ below $10^5$, there are no examples with $t_1, \dots, t_m$ all being positive. $\endgroup$ Aug 10, 2015 at 15:50

Your Answer

By clicking “Post Your Answer”, you agree to our terms of service and acknowledge you have read our privacy policy.

Not the answer you're looking for? Browse other questions tagged or ask your own question.